A curious prime counting approximation or just data overfitting?Is Li(x) the best possible approximation to the prime-counting function?Estimate on the prime-counting function $psi(x)$.Asymptotic bounds on $pi^-1(x)$ (inverse prime counting function)Prime-Counting Functiona question for the prime counting functionThe shortest interval for which the prime number theorem holdsParity of the Prime Counting Functionprime counting function pi boundsnth prime better approximationPrime counting. Meissel, Lehmer: is there a general formula?

A curious prime counting approximation or just data overfitting?


Is Li(x) the best possible approximation to the prime-counting function?Estimate on the prime-counting function $psi(x)$.Asymptotic bounds on $pi^-1(x)$ (inverse prime counting function)Prime-Counting Functiona question for the prime counting functionThe shortest interval for which the prime number theorem holdsParity of the Prime Counting Functionprime counting function pi boundsnth prime better approximationPrime counting. Meissel, Lehmer: is there a general formula?













3












$begingroup$


I am not sure, if this is a research problem. If not I will move this question to ME:
Let $Omega(n) = sum_p v_p(n)$, which we might view as a random variable.
Let $E_n = frac1n sum_k=1^nOmega(k)$ be the expected value and $V_n=frac1n sum_k=1^n(E_n-Omega(k))^2$ be the variance.
Then
$$pi(n) approx fracngamma(fracV_nE_n,1.4854177cdot fracV_nE_n^2)Gamma(fracV_nE_n)$$
where $Gamma=$ Gamma function, $gamma=$ lower incomplete gamma function.



Background:
I was trying to fit the gamma distribution to the random variable $Omega(k)$ ,$1 le k le n$. The value $1.4854177$ is fitted to some data.
My question is, if there is any heuristic why this approximation should be good, if at all, or if this is just an overfitting problem?



Below you can find some sage code which implements this:



def Omega(n):
return sum([valuation(n,p) for p in prime_divisors(n)])

means = []
variances = []
xxs = []
omegas = [Omega(k) for k in range(1,10^4)]
for nn in range(10^4,10^4+3*10^3+1):
n = nn
omegas.append(Omega(n))
print "---"
m = mean(omegas[1:-1])
v = variance(omegas[1:-1])
shape,scale = m^2/v,v/m
xx = find_root(lambda xx : n*(lower_gamma(shape,xx*1/scale)/gamma(shape) ).N()-prime_pi(n),1,2)
xx = 1.4854177706344873
approxPrimePi2 = n*(lower_gamma(shape,xx*1/scale)/gamma(shape) ).N()
primepi = prime_pi(n)
print primepi, approxPrimePi2,shape.N(),scale.N(),xx
print "---"
print "err2 = %s" % (abs(primepi-approxPrimePi2)/primepi)
xxs.append(xx)
means.append(m.N())
variances.append(v.N())









share|cite|improve this question











$endgroup$











  • $begingroup$
    Does $1.485...$ correspond to some known constant?
    $endgroup$
    – Sylvain JULIEN
    7 hours ago











  • $begingroup$
    I am not sure. Maybe.
    $endgroup$
    – orgesleka
    7 hours ago















3












$begingroup$


I am not sure, if this is a research problem. If not I will move this question to ME:
Let $Omega(n) = sum_p v_p(n)$, which we might view as a random variable.
Let $E_n = frac1n sum_k=1^nOmega(k)$ be the expected value and $V_n=frac1n sum_k=1^n(E_n-Omega(k))^2$ be the variance.
Then
$$pi(n) approx fracngamma(fracV_nE_n,1.4854177cdot fracV_nE_n^2)Gamma(fracV_nE_n)$$
where $Gamma=$ Gamma function, $gamma=$ lower incomplete gamma function.



Background:
I was trying to fit the gamma distribution to the random variable $Omega(k)$ ,$1 le k le n$. The value $1.4854177$ is fitted to some data.
My question is, if there is any heuristic why this approximation should be good, if at all, or if this is just an overfitting problem?



Below you can find some sage code which implements this:



def Omega(n):
return sum([valuation(n,p) for p in prime_divisors(n)])

means = []
variances = []
xxs = []
omegas = [Omega(k) for k in range(1,10^4)]
for nn in range(10^4,10^4+3*10^3+1):
n = nn
omegas.append(Omega(n))
print "---"
m = mean(omegas[1:-1])
v = variance(omegas[1:-1])
shape,scale = m^2/v,v/m
xx = find_root(lambda xx : n*(lower_gamma(shape,xx*1/scale)/gamma(shape) ).N()-prime_pi(n),1,2)
xx = 1.4854177706344873
approxPrimePi2 = n*(lower_gamma(shape,xx*1/scale)/gamma(shape) ).N()
primepi = prime_pi(n)
print primepi, approxPrimePi2,shape.N(),scale.N(),xx
print "---"
print "err2 = %s" % (abs(primepi-approxPrimePi2)/primepi)
xxs.append(xx)
means.append(m.N())
variances.append(v.N())









share|cite|improve this question











$endgroup$











  • $begingroup$
    Does $1.485...$ correspond to some known constant?
    $endgroup$
    – Sylvain JULIEN
    7 hours ago











  • $begingroup$
    I am not sure. Maybe.
    $endgroup$
    – orgesleka
    7 hours ago













3












3








3





$begingroup$


I am not sure, if this is a research problem. If not I will move this question to ME:
Let $Omega(n) = sum_p v_p(n)$, which we might view as a random variable.
Let $E_n = frac1n sum_k=1^nOmega(k)$ be the expected value and $V_n=frac1n sum_k=1^n(E_n-Omega(k))^2$ be the variance.
Then
$$pi(n) approx fracngamma(fracV_nE_n,1.4854177cdot fracV_nE_n^2)Gamma(fracV_nE_n)$$
where $Gamma=$ Gamma function, $gamma=$ lower incomplete gamma function.



Background:
I was trying to fit the gamma distribution to the random variable $Omega(k)$ ,$1 le k le n$. The value $1.4854177$ is fitted to some data.
My question is, if there is any heuristic why this approximation should be good, if at all, or if this is just an overfitting problem?



Below you can find some sage code which implements this:



def Omega(n):
return sum([valuation(n,p) for p in prime_divisors(n)])

means = []
variances = []
xxs = []
omegas = [Omega(k) for k in range(1,10^4)]
for nn in range(10^4,10^4+3*10^3+1):
n = nn
omegas.append(Omega(n))
print "---"
m = mean(omegas[1:-1])
v = variance(omegas[1:-1])
shape,scale = m^2/v,v/m
xx = find_root(lambda xx : n*(lower_gamma(shape,xx*1/scale)/gamma(shape) ).N()-prime_pi(n),1,2)
xx = 1.4854177706344873
approxPrimePi2 = n*(lower_gamma(shape,xx*1/scale)/gamma(shape) ).N()
primepi = prime_pi(n)
print primepi, approxPrimePi2,shape.N(),scale.N(),xx
print "---"
print "err2 = %s" % (abs(primepi-approxPrimePi2)/primepi)
xxs.append(xx)
means.append(m.N())
variances.append(v.N())









share|cite|improve this question











$endgroup$




I am not sure, if this is a research problem. If not I will move this question to ME:
Let $Omega(n) = sum_p v_p(n)$, which we might view as a random variable.
Let $E_n = frac1n sum_k=1^nOmega(k)$ be the expected value and $V_n=frac1n sum_k=1^n(E_n-Omega(k))^2$ be the variance.
Then
$$pi(n) approx fracngamma(fracV_nE_n,1.4854177cdot fracV_nE_n^2)Gamma(fracV_nE_n)$$
where $Gamma=$ Gamma function, $gamma=$ lower incomplete gamma function.



Background:
I was trying to fit the gamma distribution to the random variable $Omega(k)$ ,$1 le k le n$. The value $1.4854177$ is fitted to some data.
My question is, if there is any heuristic why this approximation should be good, if at all, or if this is just an overfitting problem?



Below you can find some sage code which implements this:



def Omega(n):
return sum([valuation(n,p) for p in prime_divisors(n)])

means = []
variances = []
xxs = []
omegas = [Omega(k) for k in range(1,10^4)]
for nn in range(10^4,10^4+3*10^3+1):
n = nn
omegas.append(Omega(n))
print "---"
m = mean(omegas[1:-1])
v = variance(omegas[1:-1])
shape,scale = m^2/v,v/m
xx = find_root(lambda xx : n*(lower_gamma(shape,xx*1/scale)/gamma(shape) ).N()-prime_pi(n),1,2)
xx = 1.4854177706344873
approxPrimePi2 = n*(lower_gamma(shape,xx*1/scale)/gamma(shape) ).N()
primepi = prime_pi(n)
print primepi, approxPrimePi2,shape.N(),scale.N(),xx
print "---"
print "err2 = %s" % (abs(primepi-approxPrimePi2)/primepi)
xxs.append(xx)
means.append(m.N())
variances.append(v.N())






nt.number-theory analytic-number-theory prime-numbers prime-number-theorem






share|cite|improve this question















share|cite|improve this question













share|cite|improve this question




share|cite|improve this question








edited 7 hours ago









GH from MO

60.8k5153232




60.8k5153232










asked 8 hours ago









orgeslekaorgesleka

724417




724417











  • $begingroup$
    Does $1.485...$ correspond to some known constant?
    $endgroup$
    – Sylvain JULIEN
    7 hours ago











  • $begingroup$
    I am not sure. Maybe.
    $endgroup$
    – orgesleka
    7 hours ago
















  • $begingroup$
    Does $1.485...$ correspond to some known constant?
    $endgroup$
    – Sylvain JULIEN
    7 hours ago











  • $begingroup$
    I am not sure. Maybe.
    $endgroup$
    – orgesleka
    7 hours ago















$begingroup$
Does $1.485...$ correspond to some known constant?
$endgroup$
– Sylvain JULIEN
7 hours ago





$begingroup$
Does $1.485...$ correspond to some known constant?
$endgroup$
– Sylvain JULIEN
7 hours ago













$begingroup$
I am not sure. Maybe.
$endgroup$
– orgesleka
7 hours ago




$begingroup$
I am not sure. Maybe.
$endgroup$
– orgesleka
7 hours ago










1 Answer
1






active

oldest

votes


















8












$begingroup$

Your heuristic approximation is not correct. It was proved by Turán (1934) that
$E_n$ and $V_n$ are both asymptotically $loglog n$. As a result, the RHS of your display is
$$nfracgammaleft(1+o(1),frac1.4854177+o(1)loglog nright)Gammabigl(1+o(1)bigr)=nfrac1.4854177+o(1)loglog n.$$
On the other hand, $pi(n)$ is asymptotically $n/log n$ by the prime number theorem.






share|cite|improve this answer











$endgroup$








  • 1




    $begingroup$
    thanks. i thought that there must be some mistake
    $endgroup$
    – orgesleka
    7 hours ago











Your Answer








StackExchange.ready(function()
var channelOptions =
tags: "".split(" "),
id: "504"
;
initTagRenderer("".split(" "), "".split(" "), channelOptions);

StackExchange.using("externalEditor", function()
// Have to fire editor after snippets, if snippets enabled
if (StackExchange.settings.snippets.snippetsEnabled)
StackExchange.using("snippets", function()
createEditor();
);

else
createEditor();

);

function createEditor()
StackExchange.prepareEditor(
heartbeatType: 'answer',
autoActivateHeartbeat: false,
convertImagesToLinks: true,
noModals: true,
showLowRepImageUploadWarning: true,
reputationToPostImages: 10,
bindNavPrevention: true,
postfix: "",
imageUploader:
brandingHtml: "Powered by u003ca class="icon-imgur-white" href="https://imgur.com/"u003eu003c/au003e",
contentPolicyHtml: "User contributions licensed under u003ca href="https://creativecommons.org/licenses/by-sa/3.0/"u003ecc by-sa 3.0 with attribution requiredu003c/au003e u003ca href="https://stackoverflow.com/legal/content-policy"u003e(content policy)u003c/au003e",
allowUrls: true
,
noCode: true, onDemand: true,
discardSelector: ".discard-answer"
,immediatelyShowMarkdownHelp:true
);



);













draft saved

draft discarded


















StackExchange.ready(
function ()
StackExchange.openid.initPostLogin('.new-post-login', 'https%3a%2f%2fmathoverflow.net%2fquestions%2f333258%2fa-curious-prime-counting-approximation-or-just-data-overfitting%23new-answer', 'question_page');

);

Post as a guest















Required, but never shown

























1 Answer
1






active

oldest

votes








1 Answer
1






active

oldest

votes









active

oldest

votes






active

oldest

votes









8












$begingroup$

Your heuristic approximation is not correct. It was proved by Turán (1934) that
$E_n$ and $V_n$ are both asymptotically $loglog n$. As a result, the RHS of your display is
$$nfracgammaleft(1+o(1),frac1.4854177+o(1)loglog nright)Gammabigl(1+o(1)bigr)=nfrac1.4854177+o(1)loglog n.$$
On the other hand, $pi(n)$ is asymptotically $n/log n$ by the prime number theorem.






share|cite|improve this answer











$endgroup$








  • 1




    $begingroup$
    thanks. i thought that there must be some mistake
    $endgroup$
    – orgesleka
    7 hours ago















8












$begingroup$

Your heuristic approximation is not correct. It was proved by Turán (1934) that
$E_n$ and $V_n$ are both asymptotically $loglog n$. As a result, the RHS of your display is
$$nfracgammaleft(1+o(1),frac1.4854177+o(1)loglog nright)Gammabigl(1+o(1)bigr)=nfrac1.4854177+o(1)loglog n.$$
On the other hand, $pi(n)$ is asymptotically $n/log n$ by the prime number theorem.






share|cite|improve this answer











$endgroup$








  • 1




    $begingroup$
    thanks. i thought that there must be some mistake
    $endgroup$
    – orgesleka
    7 hours ago













8












8








8





$begingroup$

Your heuristic approximation is not correct. It was proved by Turán (1934) that
$E_n$ and $V_n$ are both asymptotically $loglog n$. As a result, the RHS of your display is
$$nfracgammaleft(1+o(1),frac1.4854177+o(1)loglog nright)Gammabigl(1+o(1)bigr)=nfrac1.4854177+o(1)loglog n.$$
On the other hand, $pi(n)$ is asymptotically $n/log n$ by the prime number theorem.






share|cite|improve this answer











$endgroup$



Your heuristic approximation is not correct. It was proved by Turán (1934) that
$E_n$ and $V_n$ are both asymptotically $loglog n$. As a result, the RHS of your display is
$$nfracgammaleft(1+o(1),frac1.4854177+o(1)loglog nright)Gammabigl(1+o(1)bigr)=nfrac1.4854177+o(1)loglog n.$$
On the other hand, $pi(n)$ is asymptotically $n/log n$ by the prime number theorem.







share|cite|improve this answer














share|cite|improve this answer



share|cite|improve this answer








edited 7 hours ago

























answered 7 hours ago









GH from MOGH from MO

60.8k5153232




60.8k5153232







  • 1




    $begingroup$
    thanks. i thought that there must be some mistake
    $endgroup$
    – orgesleka
    7 hours ago












  • 1




    $begingroup$
    thanks. i thought that there must be some mistake
    $endgroup$
    – orgesleka
    7 hours ago







1




1




$begingroup$
thanks. i thought that there must be some mistake
$endgroup$
– orgesleka
7 hours ago




$begingroup$
thanks. i thought that there must be some mistake
$endgroup$
– orgesleka
7 hours ago

















draft saved

draft discarded
















































Thanks for contributing an answer to MathOverflow!


  • Please be sure to answer the question. Provide details and share your research!

But avoid


  • Asking for help, clarification, or responding to other answers.

  • Making statements based on opinion; back them up with references or personal experience.

Use MathJax to format equations. MathJax reference.


To learn more, see our tips on writing great answers.




draft saved


draft discarded














StackExchange.ready(
function ()
StackExchange.openid.initPostLogin('.new-post-login', 'https%3a%2f%2fmathoverflow.net%2fquestions%2f333258%2fa-curious-prime-counting-approximation-or-just-data-overfitting%23new-answer', 'question_page');

);

Post as a guest















Required, but never shown





















































Required, but never shown














Required, but never shown












Required, but never shown







Required, but never shown

































Required, but never shown














Required, but never shown












Required, but never shown







Required, but never shown







Popular posts from this blog

Invision Community Contents History See also References External links Navigation menuProprietaryinvisioncommunity.comIPS Community ForumsIPS Community Forumsthis blog entry"License Changes, IP.Board 3.4, and the Future""Interview -- Matt Mecham of Ibforums""CEO Invision Power Board, Matt Mecham Is a Liar, Thief!"IPB License Explanation 1.3, 1.3.1, 2.0, and 2.1ArchivedSecurity Fixes, Updates And Enhancements For IPB 1.3.1Archived"New Demo Accounts - Invision Power Services"the original"New Default Skin"the original"Invision Power Board 3.0.0 and Applications Released"the original"Archived copy"the original"Perpetual licenses being done away with""Release Notes - Invision Power Services""Introducing: IPS Community Suite 4!"Invision Community Release Notes

Canceling a color specificationRandomly assigning color to Graphics3D objects?Default color for Filling in Mathematica 9Coloring specific elements of sets with a prime modified order in an array plotHow to pick a color differing significantly from the colors already in a given color list?Detection of the text colorColor numbers based on their valueCan color schemes for use with ColorData include opacity specification?My dynamic color schemes

Tom Holland Mục lục Đầu đời và giáo dục | Sự nghiệp | Cuộc sống cá nhân | Phim tham gia | Giải thưởng và đề cử | Chú thích | Liên kết ngoài | Trình đơn chuyển hướngProfile“Person Details for Thomas Stanley Holland, "England and Wales Birth Registration Index, 1837-2008" — FamilySearch.org”"Meet Tom Holland... the 16-year-old star of The Impossible""Schoolboy actor Tom Holland finds himself in Oscar contention for role in tsunami drama"“Naomi Watts on the Prince William and Harry's reaction to her film about the late Princess Diana”lưu trữ"Holland and Pflueger Are West End's Two New 'Billy Elliots'""I'm so envious of my son, the movie star! British writer Dominic Holland's spent 20 years trying to crack Hollywood - but he's been beaten to it by a very unlikely rival"“Richard and Margaret Povey of Jersey, Channel Islands, UK: Information about Thomas Stanley Holland”"Tom Holland to play Billy Elliot""New Billy Elliot leaving the garage"Billy Elliot the Musical - Tom Holland - Billy"A Tale of four Billys: Tom Holland""The Feel Good Factor""Thames Christian College schoolboys join Myleene Klass for The Feelgood Factor""Government launches £600,000 arts bursaries pilot""BILLY's Chapman, Holland, Gardner & Jackson-Keen Visit Prime Minister""Elton John 'blown away' by Billy Elliot fifth birthday" (video with John's interview and fragments of Holland's performance)"First News interviews Arrietty's Tom Holland"“33rd Critics' Circle Film Awards winners”“National Board of Review Current Awards”Bản gốc"Ron Howard Whaling Tale 'In The Heart Of The Sea' Casts Tom Holland"“'Spider-Man' Finds Tom Holland to Star as New Web-Slinger”lưu trữ“Captain America: Civil War (2016)”“Film Review: ‘Captain America: Civil War’”lưu trữ“‘Captain America: Civil War’ review: Choose your own avenger”lưu trữ“The Lost City of Z reviews”“Sony Pictures and Marvel Studios Find Their 'Spider-Man' Star and Director”“‘Mary Magdalene’, ‘Current War’ & ‘Wind River’ Get 2017 Release Dates From Weinstein”“Lionsgate Unleashing Daisy Ridley & Tom Holland Starrer ‘Chaos Walking’ In Cannes”“PTA's 'Master' Leads Chicago Film Critics Nominations, UPDATED: Houston and Indiana Critics Nominations”“Nominaciones Goya 2013 Telecinco Cinema – ENG”“Jameson Empire Film Awards: Martin Freeman wins best actor for performance in The Hobbit”“34th Annual Young Artist Awards”Bản gốc“Teen Choice Awards 2016—Captain America: Civil War Leads Second Wave of Nominations”“BAFTA Film Award Nominations: ‘La La Land’ Leads Race”“Saturn Awards Nominations 2017: 'Rogue One,' 'Walking Dead' Lead”Tom HollandTom HollandTom HollandTom Hollandmedia.gettyimages.comWorldCat Identities300279794no20130442900000 0004 0355 42791085670554170004732cb16706349t(data)XX5557367